interestedintacos
Thanks Received: 58
Atticus Finch
Atticus Finch
 
Posts: 116
Joined: November 09th, 2010
 
 
trophy
Most Thanked
trophy
First Responder
 

Q3 - forbidding companies from hiring permanent-flawed quest

by interestedintacos Tue Apr 12, 2011 8:01 pm

Can someone explain why D is less supported than C? I'm really having trouble here.

I guess that C is the sort of "everyday logic" answer that they are probably looking for out of the third question in a section. But because of the way it's worded it seems to me that picking choice C means interpreting "sufficient" to mean "more," an unsupported inference.

On the other hand, if we take "sufficient" to only mean sufficient choice D is supported.

David says: it's unfair to forbid companies from hiring permanent replacements because it would lead to companies having little leverage in negotiations with strikers.

Lin says: "no," i.e. it is not unfair, because regardless of any reduction companies would still have sufficient leverage if they hired temporary workers.

Choice D says that the disagreement is based on how much leverage companies should have. David thinks it's unfair, thus wrong, for companies to end up with "little leverage." Lin says that's not true because the companies could still have "sufficient" leverage." That's my interpretation.

Choice C says they disagree over the amount lost as a result of not being able to hire permanent replacements. David of course says the amount would be big because companies end up with little (although this is actually a big inference because even though he says they end up with little that doesn't mean hiring permanent replacements would have given them HUGE leverage and not being able to hire is therefore is a BIG LOSS--it could be that they have a medium amount and the inability to hire permanent replacements makes them go from a medium amount to "a little.")

Regardless of that, Lin simply says that the companies would still have sufficient leverage anyway. For C to be correct we have to assume that by saying "sufficient" Lin is saying the amount lost is small; on the other hand, he could be saying that the amount lost is big, but that nevertheless it's still sufficient.

Can someone point out where I've gone wrong here? The only thing I see is maybe the "unfair"=wrong issue. But even given that my qualms with C seem pretty serious--I don't see how we can go from Lin saying "the companies would still have sufficient leverage" to Lin saying, as choice C would require, that the loss is small (or at least smaller than David thinks). It would require an assumption regarding what sufficient means to Lin. And that clearly seems like a value judgment regarding how much leverage he thinks companies should have against striking employees.
 
giladedelman
Thanks Received: 833
LSAT Geek
 
Posts: 619
Joined: April 04th, 2010
 
 
 

Re: Q3 - forbidding companies from hiring permanent-flawed quest

by giladedelman Thu Apr 14, 2011 1:23 pm

Good post, great username. :D

Your approach to this question is really solid -- we need to pick an answer that we can prove David and Lin would disagree about. I just think you're getting a little twisted around on the details.

For starters, do we know that Lin disagrees with David's conclusion? That is, do we know that "no" means "it's not unfair"? We do not know that; it's ambiguous. "No" could mean "it's not unfair," or it could mean "they wouldn't have little leverage." The only thing we know for sure is that Lin believes in the latter proposition, because he follows up by explicitly saying that they would still have sufficient leverage.

And that last part is the key. David believes the companies would have little leverage; Lin says no, they would still have sufficient leverage. So it doesn't matter exactly how much "sufficient" means, because it is clear that it is more than "little." In other words, David thinks companies would lose so much leverage as to end up with too little, but Lin thinks they could still end up with enough -- so Lin doesn't think they would lose as much leverage. That's why (C) is correct.

The problem with (D) is that Lin has no opinion on how much leverage the companies should have. He only weighs in on how much leverage the companies would have.

Your explanation suggests that Lin and David agree on how much leverage the companies would have, and disagree about whether that's fair or not. But the disagreement is actually about the amount of leverage: is it little, or is it sufficient?

(Again, we don't have to worry about whether little could equal sufficient, because Lin makes it clear that he considers them to be different.)

And in the interest of thoroughness:

(A) is incorrect because David says they will have little leverage, not no leverage.

(B) is unsupported by both speakers: we don't know whether they think companies and employees should have equal leverage.

(E) is incorrect because we don't know whether Lin thinks it's fair or unfair; if anything, we would expect him to say it's fair, since he disagrees with David's premise.

Does that clear this one up for you?
 
interestedintacos
Thanks Received: 58
Atticus Finch
Atticus Finch
 
Posts: 116
Joined: November 09th, 2010
 
 
trophy
Most Thanked
trophy
First Responder
 

Re: Q3 - forbidding companies from hiring permanent-flawed quest

by interestedintacos Thu Apr 14, 2011 8:55 pm

Thanks for pointing out that the "no" should obviously not immediately be taken to be against the conclusion. We're supposed to analyze what comes after the "no" to see what it refers to.

Nevertheless, the problem remains for me because of the assumptions necessary for the intended LSAC interpretation to be right. First, you do have to assume that by "sufficient" Lin means more than little. The fact Lin says "sufficient" doesn't necessarily mean "more than little." You're saying we should go with that definition because it makes the most sense in the context. I would say that's true except for the fact that if you interpret the "no" to refer to the conclusion, and thus that Lin is saying the amount of leverage is fair, it would make sense for sufficient to actually have its normal definition (that the amount of leverage they'll get is sufficient whether or not it's "little").

When I considered this I thought it would be better to take "sufficient" to mean sufficient and not to mean "more than little." Thus I favored the interpretation of "no" I just mentioned. Strictly speaking if someone says "there'll only be 5 dollars left so we can't buy tacos" (analogues to "so it's unfair"), and I respond "no, what we have will be sufficient if we get a discount" ("if they hire temporary workers"), that doesn't mean I'm saying we have more than 5 dollars. I could be agreeing we have 5 bucks but simply pointing out that 5 bucks is in fact sufficient--i would be disputing the idea that what we have isn't enough money, not that we only have 5 dollars.

Perhaps it's from doing too much logic stuff, but for me when I hear "sufficient" I don't at all associate that with a higher number or "more than little." Often a little amount is more than sufficient. Nevertheless I showed the question to non-LSAT people, and they all got it right based on pure intuition without a second thought. And they all told me the exact same thing about "sufficient" meaning "more than little." So I think the lesson was, especially at this difficulty level, to not reject what may be an everyday assumption, even if it's in regard to an LSAT word like "sufficient."

The other assumption, by the way, that's required is that David actually thinks there's a sizable loss of leverage as a result of the change. He never says that. He only says that as a result of the change the companies will end up with a small amount of leverage. You have to assume he means there will be a sizeable or big reduction and that Lin is countering that.

Bottom line: I think these assumptions threw me, especially about "sufficient," but the main lesson I have taken from it is to be ready to make everyday assumptions on easier questions. This also came up in the past where on a particular question it was necessary to consider "most" to exclude the possibility of "all," which is of course at odds with the typical treatment of "most" on the LSAT.
 
giladedelman
Thanks Received: 833
LSAT Geek
 
Posts: 619
Joined: April 04th, 2010
 
This post thanked 2 times.
 
 

Re: Q3 - forbidding companies from hiring permanent-flawed quest

by giladedelman Sun Apr 17, 2011 12:27 am

What a fun discussion!

The absolute biggest takeaway you should have from this question is that we must always WORK FROM WRONG TO RIGHT. Answers A, B, D, and E are just 100% wrong. (With respect to [D], even if we interpret Lin as saying that the situation is not unfair, that doesn't tell us how much leverage he thinks companies should have. Maybe this situation would be fair, but what would really be great is if they had even more leverage.)

So even if you have some qualms about (C), it's clearly the most strongly supported statement because it is at least pretty plausible, compared to the other four, which are totally implausible.

Okay, we should probably stop there. Read on at your own peril.

I think the key to understanding why (C) is supported is that when David says companies would have "little" leverage, that plainly means that he thinks they would not have enough. It would not make sense to interpret him as saying they have little leverage, which is enough, but still unfair.

So David believes the companies would not have enough leverage. Lin believes the companies would have enough. Crucially, this is not analogous to your taco example. (You really are obsessed!) Why? Because leverage is not measured in discrete or even quantifiable units; leverage is measured in terms of what goals it allows you to achieve. So to say I have "enough" leverage is by definition to say that I have more than "not enough" leverage.

So we can say that David and Lin disagree about how much leverage the companies would have. That's pretty much equivalent to them disagreeing about how much leverage would be lost, because David is saying this rule would put them below sufficiency, while Lin is saying it would not.

Again, the point is that we can very confidently interpret David as claiming that the companies would not have enough leverage. So it's a comparison between sufficient and insufficient, not between sufficient and some value.

And again, the REAL point is that the other four answers suck, this is question 3, pick (C) and move on! :)
 
rezamza
Thanks Received: 4
Vinny Gambini
Vinny Gambini
 
Posts: 6
Joined: September 08th, 2014
 
 
 

Re: Q3 - forbidding companies from hiring permanent-flawed quest

by rezamza Sat Sep 12, 2015 7:56 pm


I know its rather late, to be posting a question for this one, and Im following the discussion and reasoning presented in this thread just fine, but Im wondering why there is not discussion of Lin's comment regarding the hiring of temporary replacements?

David states that it'd be profoundly unfair to prohibit companies from hiring permanent replacements. For this would hinder the company from having anything more than little leverage in their negotiations.

Lin replies by saying "If the company hired temporary replacements, they would still have sufficient leverage." In what way does this combat David's comment? While David's discussing the hiring of permanent employees, Lin is focused on the hiring of temporary employees. How do their comments intersect at all if this is the case?

Am I mistakenly connecting David's two statements as being connected? Is David actually saying that if the company was allowed to hire permanent replacements then they would not have such a small amount of leverage?

Thanks for the clarification!
 
scarlettng0623
Thanks Received: 0
Vinny Gambini
Vinny Gambini
 
Posts: 2
Joined: March 26th, 2016
 
 
 

Re: Q3 - forbidding companies from hiring permanent-flawed quest

by scarlettng0623 Thu Aug 04, 2016 4:56 am

David and Lin are talking about two different things.

David talked about the leverage for hiring permanent employees. Lin is talking about leverage about hiring temporary employees.

How do they intersect with each other? Lin might agree with David that hiring permanent employees is too much leverage loss, which in the end it shows no disagreement. Lin seems to propose an alternative. I chose (E).

Thoughts?